From bbrock@pepperdine.eduThu Aug 29 15:53:00 1996 Date: Thu, 29 Aug 1996 08:27:04 -0400 From: Bradley Brock To: NMBRTHRY@listserv.nodak.edu Subject: Re: x^a + y^b = z^c > Conjecture: > > X^a + Y^b = Z^c has no solutions for coprime naturals X,Y,Z when each > exponent a,b,c > 2. To answer this question one need only quote Granville's post to this list from 2 1/2 years ago. The bottom line is that there are no known solutions and that for fixed a, b, & c, there can be at most finitely many. Brad -- From: Andrew Granville Subject: x^p+y^q=z^r Comments: To: NmbrThry@vm1.nodak.edu To: Multiple recipients of list NMBRTHRY In reply to a question of Dave Davis on x^p+y^q=z^r: Henri Darmon and I have been looking at integer solutions to the above equations. It is trivial to find lots for many different p,q,r. For example, the following parametric solution exists for exponents p,p,p+1: (ac)^p+(bc)^p = c^(p+1) where c = a^p+b^p. To rid us of such upstart solutions, let us restrict ourselves to solutions where x and y are coprime -- what we will call `proper solutions'. It is `well-known'(see Dickson) that 1) The above equation has inf many proper solns whenever 1/p+1/q+1/r>1. 2) The above equation has no proper solns whenever 1/p+1/q+1/r=1 except 3^2-2^3=1^6 What Darmon and I have proved is: If 1/p+1/q+1/r < 1 then there are only finitely many proper solutions to x^p+y^q=z^r. The main tool we use is Faltings' theorem (nee Mordell's conjecture); which we apply via an appropriate descent argument. A preprint will soon be available (send email to me for this preprint). Small examples of proper solutions to x^p+y^q=z^r with 1/p+1/q+1/r < 1 are: 1+2^3=3^2, \ \ 2^5+7^2=3^4, \ \ 7^3 + 13^2 = 2^9, \ \ 2^7+17^3=71^2, \ \ 3^5+11^4=122^2 Extraordinarily large solutions have been found recently by Beukers and Zagier: 17^7 + 76271^3 = 21063928^2, \ \ 1414^3 + 2213459^2 = 65^7, \ \ 9262^3 + 15312283^2 = 113^7, \ \ 3^8 + 96222^3 = 30042907^2, \ \ 33^8 + 1549034^2 = 15613^3. It amazes me that there is such a gap between the two sets of solutions. It may be that these are all of the solutions to the above, but I thought that before Beukers and Zagier got involved, so maybe I am wrong again. Can anyone compute some new examples ? That would be very interesting. Andrew Granville andrew@sophie.math.uga.edu